Solving Laplace Transforms w/ Green Functions

Click For Summary
The discussion focuses on solving the differential equation for the Green's function using Laplace transforms. The equation involves the second derivative of G(t, t') and a delta function, with initial conditions set to zero. Participants highlight the need to apply Laplace transforms to both sides of the equation, leading to the transformation of the delta function. The solution is divided into two cases based on the relationship between t and t', resulting in specific forms for G(t, t') in each case. The conversation emphasizes the importance of understanding the continuity and derivative conditions at t = t' to fully determine the constants in the solution.
mateomy
Messages
305
Reaction score
0
Solve...
<br /> \frac{d^2}{dt^2}G(t,t&#039;) + \omega^2G(t,t&#039;) = \delta(t-t&#039;)<br />
Solve (above) if G=0 and \frac{dG}{dt}=0 at t=0 to obtain:
<br /> G(t,t&#039;)=\begin{cases}<br /> 0 &amp; 0&lt;t&lt;t&#039; \\<br /> \frac{1}{\omega}\sin\omega(t-t&#039;) &amp; 0&lt;t&#039;&lt;t<br /> <br /> \end{cases}<br />I'm supposed to use Laplace Transforms to figure this out. (I'm going out of Boas Chapter 8, Section 12 problem 1) To be honest, I'm having a really difficult time getting my head around Green Functions so this is really pushing on me.

The rest of the problem states, "Use laplace transforms to find the inverse transform".

I don't even know where to begin.
 
Last edited:
Physics news on Phys.org
Well, you know what a Laplace transform is, I'm assuming. Do you know the rules for evaluating the Laplace transforms of derivatives,

$$\mathcal L\left[\frac{d^nf}{dt^n}\right],$$

in terms of ##\mathcal L[f]##, for any function f(t)?

What happens if you apply the Laplace transform to your differential equation? i.e., what is

$$\mathcal L\left[ \frac{d^2}{dt^2}G(t,t') + \omega^2 G(t,t')\right]$$
in terms of the Laplace transform of ##G(t,t')##? (transforming with respect to t, not t').

Once you've found that, you know it must also be equal to the Laplace transform of the delta function, so you will then need to solve for ##\mathcal L[G]## and inverse transform to get G(t,t').
 
Hmmm, I'm rereading the Laplace section carefully. I'll reaffirm that and then I'll try your hints. I'll come back with potential/probable issues. Thank you.
 
Personally, I have never liked "Laplace Transform methods"- they hide too much in the machinery. Here's how I would approach this problem:

Write G_1(t, t&#039;) for G(t, t') for 0<t< t' and G_2(t, t&#039;) for G(t, t') for t'< t< 1.

For 0< t< t', G_1(t, t&#039;) must satisfy d^2/dt^2G_1+ \omega^2G_1= 0 so G_1(t, t&#039;)= A cos(\omega t)+ B sin(\omega t). Since the initial conditions are G(0, t')= 0, dG(0, t')/dt= 0, A= B= 0 so G_1(t, t&#039;)= 0 for all 0&lt; t&lt; t&#039;.<br /> <br /> For t&#039;&lt; t&lt; 1, G_2(t, t&amp;#039;) must satisfy the same differential equation so that G_2(t, t&amp;#039;)= A cos(\omega t)+ B sin(\omega t). To find the &quot;initial conditions&quot; we use the facts that the Green&#039;s function is continuous at t= t&#039; and has a unit jump discontinuity in the first derivative at t= t&#039;.<br /> <br /> So G_2(t&amp;#039;, t&amp;#039;)= Acos(\omega t&amp;#039;)+ B sin(\omega t&amp;#039;)= 0 and dG(t&amp;#039;,t&amp;#039;)/dt= -\omega A sin(\omega t&amp;#039;)+ \omega B cos(\omega t&amp;#039;)= 0. That gives two equations to solve for A and B in terms of t&#039;.
 
Question: A clock's minute hand has length 4 and its hour hand has length 3. What is the distance between the tips at the moment when it is increasing most rapidly?(Putnam Exam Question) Answer: Making assumption that both the hands moves at constant angular velocities, the answer is ## \sqrt{7} .## But don't you think this assumption is somewhat doubtful and wrong?

Similar threads

  • · Replies 1 ·
Replies
1
Views
1K
  • · Replies 3 ·
Replies
3
Views
3K
Replies
2
Views
2K
  • · Replies 1 ·
Replies
1
Views
2K
  • · Replies 4 ·
Replies
4
Views
3K
Replies
1
Views
2K
  • · Replies 3 ·
Replies
3
Views
1K
  • · Replies 4 ·
Replies
4
Views
3K
  • · Replies 1 ·
Replies
1
Views
1K
  • · Replies 8 ·
Replies
8
Views
2K